A square has a perimeter of 80 m. What is the length of each side?

Answers

Answer 1

Answer:

20 m

[tex]p = 4a \: thus \: a = p \div 4 = 80 \div 4 = 20 \: m[/tex]

Answer 2

Answer:

20

Step-by-step explanation:

P=4L

80=4L

L=80/4

L=20m


Related Questions

Vectors a = (1,0) and b= (1,1) are given. For what λ vector a + λb is perpendicular to vector a?

Answers

Answer:

-1

Step-by-step explanation:

Vector a=(1,0) if visualized as a line has points (0,0) and (1,0). This means the line is horizontal and has equation y=0.

This means we are looking for a vertical line.

Instead of lambda, I'm going to use u.

So we want a+ub=(1,0)+u(1,1). to vertical. If we visualize this as a line with points (0,0) and (1+u,u). We would need u=-1 so this would be vertical.

So vectors a=(1,0) and a+ub=(1,0)+u(1,1) are perpendicular for u=-1.

So we are saying vectors (1,0) and (0,-1) are perpendicular. If you draw these, you can get a visual confirmation.

The value of λ is -1 and is perpendicular to vector a.

What is Vector?

A vector is a number or phenomena with two distinct properties: magnitude and direction. The term can also refer to a quantity's mathematical or geometrical representation. In nature, vectors include velocity, momentum, force, electromagnetic fields, and weight.

We have,

Vector a = (1, 0) and b = (1, 1).

Here the vector is visualized as line is horizontal and has equation y=0.

Now, we have a + λb is perpendicular to vector a.

So, a + λb = (1, 0) + λ(1, 1)

                 = (1 + λ, 1)

Now, to get the vertical component we must have λ= -1.

Then, a(1, 0) and a+λb= (1,0)+ λ(1, ) are perpendicular to λ=-1.

Learn more about Vector here:

https://brainly.com/question/29740341

#SPJ3

Which expressions are equivalent to 5+(-3)(6x-5)5+(−3)(6x−5)5, plus, left parenthesis, minus, 3, right parenthesis, left parenthesis, 6, x, minus, 5, right parenthesis ? Choose all answers that apply: Choose all answers that apply: (Choice A) A 18x-2018x−2018, x, minus, 20 (Choice B) B 3x-33x−33, x, minus, 3 (Choice C) C None of the above

Answers

5+(-18x+15)5 + (-18x +15)5

5+ (-90x +75) + (-90x + 75)

-180x +155

let's see the alternatives:

a) -2000x - 2018 -> -200x - 201,8

b) -30x - 33 -> -180x -198

c) none of the above

so, c is the right one

hope it helps :)

Answer:

ITS C

Step-by-step explanation:

Please help me out with my maths I would really appreciate it ​

Answers

Answer:

a.

1. The rule in this sequence is (+5) every next pattern

2. 22, 27, 32, 37

3. 42

4. 12 term

5. 67

b.

1. The rule in this sequence is (-3) every next pattern

2. -7, -10, -13, -16

3. -16

4. 13 term

5. -37

Step-by-step explanation:

Answer:

I'm not sure about the answer.

a.

1. The rule in this sequence is (+5) every next pattern

2. 22, 27, 32, 37

3. 42

4. 12 term

5. 67

b.

1. The rule in this sequence is (-3) every next pattern

2. -7, -10, -13, -16

3. -16

4. 13 term

5. -37

Hope this helps you ^^

An employee is scheduled to work 40 hours per week at a base rate of $18.50 per hour. In addition to the 40 scheduled hours, the employee is asked to work 8 hours of overtime for one week due to staffing shortages. If overtime is paid at a rate of 1.5 times the base rate, how much will the employee earn for the week? (use $xxx.Xx format)

Answers

Answer:

$962.00

Step-by-step explanation:

We can start from the first sentence of the word problem and work from there.

First, the employee is working for 40 hours at $18.50 per hour. This means that for each hour, the employee is gaining $18.50 . This can be represented as $18.50 * 40 = $740 for their base pay.

Next, the employee works 8 hours of overtime at 1.5 * base pay (18.50). For each hour of overtime they work, they earn 1.5 * 18.50 = $27.75 dollars. Their earnings from overtime work for the week can be represented as

8 * 27.75 = $222

Because all the employee's hours are encompassed in overtime and base pay, we can add the two together to get

740 + 222 = $962.00 for their total pay for the week

Solve for the value of n.
n =

Answers

Answer:

136+(4n-8)=180

136+4n-8=180

4n+128=180

4n=52

n=13

Step-by-step explanation:

please mark me as brainliest

The answer is n=13 hope that helps

[(-33) + 46 + 68] - |15 - 61|

Answers

Answer:

127

Step-by-step explanation:

[(-33) + 46 + 68] - (15 - 61)

(114-33)-(-46)

81+46

127

find the missing side​

Answers

Answer:

x ≈ 13.7

Step-by-step explanation:

Using the cosine ratio in the right triangle

cos70° = [tex]\frac{adjacent}{hypotenuse}[/tex] = [tex]\frac{x}{40}[/tex] ( multiply both sides by 40 )

40 × cos70° = x , then

x ≈ 13.7 ( to the nearest tenth )

8/3=12/n solve for n

Answers

Answer:

[tex]n=\frac{9}{2}[/tex]

Step-by-step explanation:

[tex]\frac{8}{3} =\frac{12}{n}[/tex]

Cross multiply.

8n=12(3)

8n=36

[tex]n=\frac{9}{2}[/tex]

I hope this helps!

pls ❤ and give brainliest pls

Answer: n = 9/2

Step-by-step explanation:

8/3 = 12/n

Now doing cross multiplication

8/3 = 12/n

8(n) = 12(3)

8n = 36

n = 36/8

n = 9/2

Therefore value of n is 9/2

Must click thanks and mark brainliest

Question: "If y > 3, what is the value of n ?"

Answers

Answer:

y-3

Problem:

What is the remainder when the dividend is xy-3, the divisor is y, and the quotient is x-1. ?

Step-by-step explanation:

Dividend=quotient×divisor+remainder

So we have

xy-3=(x-1)×(y)+remainder

xy-3=(xy-y)+remainder *distributive property

Now we just need to figure out what polynomial goes in for the remainder so this will be a true identity.

We need to get rid of minus y so we need plus y in the remainder.

We also need minus 3 in the remainder.

So the remainder is y-3.

Let's try it out:

xy-3=(xy-y)+remainder

xy-3=(xy-y)+(y-3)

xy-3=xy-3 is what we wanted so we are done here.

please help. only need to do part b

Answers

Unless I’m reading it wrong, 20 students jumped 4 meters so 19 must have jumped less. As for more than 4.5, only 45 students jumped 4.5 meters, and only 60 students altogether, so the final 15 must have jumped more than 4.5.

Solve for x 3x+3/x-4 = 3x+2/x+4

Answers

Answer:

here you go! with step by step so you can do it next time

Answer:

-7/8

Step-by-step explanation:

cross multiply first then expand the equations when you cross multiply it will be

(3x+3)(x+4)=(3x+2)(x-1)

3x(x+4)+3(x+4)=3x(x-1)+2(x-1)

3x²+12x+3x+12=3x²-3x+2x-2

3x²+15x+12=3x²-x-2

3x²-3x²+15x+x=-2-12

16x/16=-14/16

x=-14/16

simplified to

-7/8

I hope this helps

Someone help pleaseee

Answers

Answer:

see explanation

Step-by-step explanation:

The area (A) of a rectangle is calculated as

A = length × breadth

   = (2 + [tex]\sqrt{2}[/tex] )(4-2[tex]\sqrt{2}[/tex] ) ← expand using FOIL

   = 8 - 4[tex]\sqrt{2}[/tex] + 4[tex]\sqrt{2}[/tex] - 4 ← collect like terms

   = 4 units²

--------------------------------------------------------

The opposite sides of a rectangle are congruent , so

perimeter = 2(4 - 2 [tex]\sqrt{2}[/tex]) + 2(2 + [tex]\sqrt{2}[/tex] ) ← distribute parenthesis

                 = 8 - 4[tex]\sqrt{2}[/tex] + 4 + 2[tex]\sqrt{2}[/tex] ← collect like terms

                 = 12 - 2[tex]\sqrt{2}[/tex] units

see question in image

Answers

Answer:

b) 1/9

Step-by-step explanation:

Rolling two dice, there are 6*6 = 36 outcomes

The outcomes with the difference of 4:

1&5, 2&6, 6&2, 5&1 - total of 4

Required probability:

P = 4/36 = 1/9

Correct choice is b

In circle O, and are diameters. The measure of arc AB is 55° and the measure of arc CD is 25°.

Circle O is shown. Line segments A D and B E are diameters. Line segments O F and O C are radii.

What is the measure of Arc E A C?

Answers

Your Answer Is in this attachment.

Answer:

D-212 Degrees

Step-by-step explanation

i got it right on edge

What are the solutions to the system of equations?

{y=2x²−6x+3
{y=x−2

Answers

Answer:

x = 1, y = −1

x = 5/2, y = 1/2

Step-by-step explanation:

From the question given above, the following data were obtained:

y = 2x² − 6x + 3 ........ (1)

y = x − 2 ...... (2)

We can obtain the solutions to the equation as follow:

y = 2x² − 6x + 3 ........ (1)

y = x − 2 ...... (2)

Substitute the value of y in equation 2 into equation 1

y = 2x² − 6x + 3

y = x − 2

2x² − 6x + 3 = x − 2

Rearrange

2x² − 6x − x + 3 + 2 = 0

2x² − 7x + 5 = 0

Solve by factorization

Obtain the product of 2x² and 5. The result is 10x².

Find two factors of 10x² such that their sum will result to −7x.

The factors are −2x and −5x.

Replace −7x in the equation above with −2x and −5x as shown below:

2x² − 2x − 5x + 5 = 0

2x(x − 1) − 5(x − 1) = 0

(x − 1)(2x − 5) = 0

x − 1 = 0 or 2x − 5 = 0

x = 1 or 2x = 5

x = 1 or x = 5/2

Substitute the value of x into equation 2 to obtain y

y = x − 2

x = 1

y = 1 − 2

y = −1

x = 5/2

y = x − 2

y = 5/2 − 2

y = (5 − 4)/2

y = 1/2

SUMMARY:

x = 1, y = −1

x = 5/2, y = 1/2

I need help please!!!!

Answers

a. 122.5 g

M(t) = 300(1/2)^(31/24) = 122.5

which value of g makes 26=7(g-9)+12 a true statment

Answers

Answer:

11

Step-by-step explanation:

26=7(g-9)+12

14=7(g-9)

2=g-9

g=11

What is the solution to this inequality?
-16x>-80
A. x < 5
O B. x>-5
O c. x<-5
O D. x>5

Answers

Answer:

A

Step-by-step explanation:

Divide both sides with -16. ALWAYS remember that if you divide any number with a negative number, this "< ≤ > ≥" symbols have to change to the opposite direction

You’re given two side lengths of 10 centimeters and 8 centimeters. The angle between the sides measures 40°. How many triangles can you construct using these measurements?

Answers

Answer:

1

Step-by-step explanation:

Once you have two sides and the included angle, there is only one triangle.

Answer: 1

Answer:

The answer is B. 1

Step-by-step explanation:

I hope I helped



The figure shown to the right is an isosceles triangle, and
R is the midpoint of PS.
The fig
labeled
A. Explain when it is appropriate to use the statement PT TS.
P
R
S
B. Explain when it is appropriate to use the statement PT = TS.

Answers

Answer:

We know that an isosceles triangle has 2 of its sides being equal

With R, being the midpoint of PS, we can say that

PR=RS

Noting that, with R as midpoint, we can conclude that RT is a straight line which divides angles TPR and TSR into 2 right angle triangles

Step-by-step explanation:

therefore angle at P is 45°. Angle at S also 45°

Therefore PT = TS

This is because T is 45 degrees as well as P which is also 45 degrees

angle in triangle PTS is 180 degrees

R is 90 degrees, P is 45 degrees and the whole of T is also 45 degrees(which has been split into 2)

HELP.... please??????????????

Answers

Answers:

Functions

y = -x+11y = 2x^2-6x+4y = -7

Not functions

x = 3x^2+y^2 = 81y^2 = -5x-12

=======================================================

Explanation:

A function is possible if and only if any given x input leads to exactly one y output.

For something like x^2+y^2 = 81, we can see that x = 0 leads to either y = 9 or y = -9. So this would not be a function. We would need x to pair with only y value to have it be a function.

We have the same thing going on with y^2 = -5x-12 as well.

For anything of the form x = k, where k is any real number, this is also not a function. We have one single input only and it leads to infinitely many outputs. So in a sense, this is even worse compared to the other examples.

-----------------

In summary, we have these three non-functions:

x = 3x^2+y^2 = 81y^2 = -5x-12

Everything else is a function. You can use the vertical line test as a visual way to check.

what should be added to 4.289 to get 11​

Answers

[tex]\huge\text{Hey there!}[/tex]

[tex]\large\text{We do not know the unknown number just yet so we will label it}\\\large\text{as the variable of \boxed{\bf n}}\large\text{ until we find the result of the unknown}\\\large\text{number}[/tex]

[tex]\large\text{So, your equation is now: \underline{\underline{n + 4.289 = 11}} or \underline{\underline{4.289 + n = 11}}}[/tex]

[tex]\large\textsf{n + 4.289 = 11}\\\large\text{SUBTRACT \underline{4.289} to BOTH SIDES}\\\large\text{n + 4.289 - 4.289 = 11 - 4.289}\\\large\text{CANCEL out: 4.289 - 4.289 because that gives you 0}\\\large\text{KEEP: 11 - 4.289 because that helps you get the n-value}\\\large\text{SIMPLIFY ABOVE AND YOU HAVE YOUR RESULT}\\\large\text{n = \bf 6.711}\\\\\boxed{\boxed{\huge\text{Therefore, your answer is: \bf 6.711}}}\huge\checkmark[/tex]

[tex]\huge\text{Good luck on your assignment \& enjoy your day!}[/tex]

~[tex]\frak{Amphitrite1040:)}[/tex]

Let the number which should be added is x

ATQ

[tex]\\ \sf\longmapsto x+4.289=11[/tex]

Take 4.289 to right

[tex]\\ \sf\longmapsto x=11-4.289[/tex]

[tex]\\ \sf\longmapsto x=6.711[/tex]

6.711 should be added to 4.289 to get 11

helpppppppp will mark brainlest​

Answers

Answer:

-8

Step-by-step explanation:

what do you think, when you look at the examples given in the problem definition ?

don't you see the pattern, that f(x) = x+2 ?

f(1) = 1+2 = 3

f(2) = 2+2 = 4

f(3) = 3+2 = 5

so, if we follow this assumption, then

f(-10) = -10 + 2 = -8



Which composite function can be used to find the
force of the object based on its volume?

The density of titanium is 4.5 g/cm3. A titanium object
is accelerating at a rate of 800 cm/s2. The mass of
the object can be modeled by the function m(v) =
4.5v, where v is the volume in cubic centimeters.
Additionally, the force of the object can be found
using the function F(m) = 800m.

A. F(m(v)) = 177.8V
B. F(m(v)) = 795.5v
C. F(m(v)) = 804.5v
D. F(m(V)) = 3,600V

Answers

Given:

The mass function is:

[tex]m(v)=4.5v[/tex]

where v is the volume in cubic centimeters.

The force function is:

[tex]F(m)=800m[/tex]

To find:

The composite function can be used to find the force of the object based on its volume.

Solution:

The composite function can be used to find the force of the object based on its volume is:

[tex]F(m(v))=F(4.5v)[/tex]              [tex][\because m(v)=4.5v][/tex]

[tex]F(m(v))=800(4.5v)[/tex]              [tex][\because F(m)=800m][/tex]

[tex]F(m(v))=3600v[/tex]

Therefore, the correct option is D.

Answer: F(m(v)) = 3,600v

Step-by-step explanation:DDDD



The perimeter of a rectangle is 56 feet and
its area is 192 square feet. What are the
dimensions of the rectangle?

Answers

Answer:

Step-by-step explanation:

P = 2(L + W)

Area = L*W

Area = 192

(L + W)*2 = 56

L+W = 28

L = 28 - W

W*(28 - W) = 192

28W - w^2 = 92

-w^2 + 28w - 192 = 0

w^2 - 28w + 192  = 0

This factors into

(w - 12)(w - 16) = 0

w - 12 = 0

w = 12

L = 28 - 12 = 16

Please Help I don't get this

Answers

Answer:

The choose (D)

Step-by-step explanation:

[tex] \frac{x - 16}{ {x}^{2} + 6x - 40 } + \frac{1}{x + 10} \\ = \frac{x - 16}{(x - 4)(x + 10)} + \frac{1}{x + 10} \\ = \frac{(x - 16) + (x - 4)}{(x + 10)(x - 4)} \\ = \frac{2x - 20}{(x + 10)(x - 4)} \\ = \frac{2x - 20}{ {x}^{2} + 6x - 40 } [/tex]

prove that.....cos^2α(cosec^2α-cot^2α)=cos^2α​

Answers

Step-by-step explanation:

hope this helps. ........

A sample of 50 observations is taken from an infinite population. The sampling distribution of : a.is approximately normal because of the central limit theorem. b.cannot be determined. c.is approximately normal because is always approximately normally distributed. d.is approximately normal because the sample size is small in comparison to the population size.

Answers

Answer:

a.is approximately normal because of the central limit theorem.

Step-by-step explanation:

The central limit theorem states that if we have a population with mean μ and standard deviation σ and we take sufficiently large random samples from the population with replacement, then the distribution of the sample means will be approximately normally distributed.

For any distribution if the number of samples n ≥ 30, the sample distribution will be approximately normal.

Since in our question, the sample of observations is 50, n = 50.

Since 50 > 30, then our sample distribution will be approximately normal because of the central limit theorem.

So, a is the answer.

Solve Each of the following equations:

|5x|=3

Answers

Answer:

|5x|=3  

5x=3 or 5x=-3  

divide both side by 5  

x=3/5 or -3/5

Step-by-step explanation:

Answer: X = -3/5

X = 3/5

Step-by-step explanation:

-3=5X=3

5X= -3

X= -3/5

5X = 3

X = 3/5

Maddie guessed that there were
1,905 candies in the jar.
What is the value of the 9?

Answers

Answer:

hundreths the 9 represents 900

Step-by-step explanation:

Answer:

900

Step-by-step explanation:

1,905

Expand the number

1000 + 900 + 5

900 is the value of the 9

Other Questions
analysis this please and Ill give Brainly How can our society progress ? define local autonomy I need help please thank you If the constant is added to every observation of data then arithmatic mean obtained is find the value of x. give reasons to justify your answer NEED HELP ASAP!!!! A recessed luminaire bears no marking indicating that it is ""Identified for Through- Wiring."" Is it permitted to run branch-circuit conductors other than the conductors that supply the luminaire through the integral junction box on the luminaire? ways in which soil organisms such as earth worms improve soil fertility 5. A body falls freely from rest. It covers as much distance in the last second of itsmotion as covered in the first three seconds. The body has fallen for a time of:B) 5sC) 7sD) 9sA) 35 Please help explanation if possible 1. El gnero dramtico en lengua castellana surge al final del siglo XIII. 2. Las primeras manifestaciones del gnero dramtico eran piezas que se representaban en las plazas y sitios pblicos.3. La nica obra teatral de esta poca que se conserva de forma fragmentada es La Celestina. 4. La Tragicomedia de Calisto y Melibea no es una obra representable, debido a que est escrita en forma dialogada. 5. El gnero literario y tpicamente medieval al que pertenece la obra La Celestina, es la tragicomedia. 6. Dante Alighieri fue el creador de la comedia humanstica.7. Las primeras obras teatrales en castellano que se conservan son de carcter profano.8. El teatro deja los atrios de la iglesia y se traslada a las plazas de las ciudades, como elemento de diversin del pblico.9. Una de las razones por la que La Celestina se considera una novela es por la extensin de la obra.10. El autor de La Celestina es Jorge Manrique. What should be subtracted from 13/(-56) to get 11/28 Yiyi's favorite kind of comedy is the kind that wasn't supposed to be funny in the first place. What kind of film does Yiyi love mostA. slapstick comedyB. campy comedyC. situation comedyD. physical comedy What effect will replacing x with (x - 4) have on the graph of the equation y = (x - 3) ^ 2A. Slides the graph 4 units upB. Slides the graph 7 units down C. Slides the graph 4 units right D. Slides the graph 1 units A sales associate listed a condo for $205,000. A sales associate from a competing office called the listing associate to inform him of a verbal offer of $190,000 on the property. The listing associate did not present the verbal offer. Which statement applies to this situation Help mee whats the answerr :( Q: The midpoint of the line segment with the coordinates (9, -8) and (-2, 1) is _ (f^3-5f+25)-(4f^2-12f+9) What is the inverse of the function ? f(x)=x+1 over x?plato 2- A copper wire of 3mm diameter with conductivity of 6.7 10' (0.M), and electron mobility of 0.0064 m2 /V sec. Is subjected to an electric field of 30 mV/m. Find (a) the charge density of free electrons, (b) the current density, (c) current flowing in the wire, (d) the electron draft velocity. pls help me due tomorrow